Finding the constrain of integralML-inequality for real integralsEvaluating the integral $int_0^infty fracsin x x ,mathrm dx = frac pi 2$?Proving the inequality of an integral applied to a sumFind values so that integral is a bounded operatorProblem in series expansionSolving the improper integral $1/(x^a+y^b)$Convergence of the integral: $I_alpha =int _0^infty left(frace^-alpha xsqrtxright):dx$definition of Fourier transform questionsHow to calculate $iint_D (x^2 + y^2) , dxdy$ with $D = (fracx2)^2 + (fracy3)^2 leq 1$?Integral transform with reciprocal complex exponential functions?permute double integral and limit

Traffic law UK, pedestrians

Word for a small burst of laughter that can't be held back

Why is quantum entanglement surprising?

What happens if you do emergency landing on a US base in middle of the ocean?

How do I calculate APR from monthly instalments?

How hard would it be to convert a glider into an powered electric aircraft?

Why does the Schrödinger equation work so well for the Hydrogen atom despite the relativistic boundary at the nucleus?

On the Twin Paradox Again

Building a road to escape Earth's gravity by making a pyramid on Antartica

How to supress loops in a digraph?

You've spoiled/damaged the card

What is in `tex.print` or `tex.sprint`?

When writing an error prompt, should we end the sentence with a exclamation mark or a dot?

Pronoun introduced before its antecedent

Function to extract float from different price patterns

Calling GPL'ed socket server inside Docker?

What happens to foam insulation board after you pour concrete slab?

Why don’t airliners have temporary liveries?

Avoiding cliches when writing gods

Implement Homestuck's Catenative Doomsday Dice Cascader

How bad would a partial hash leak be, realistically?

In this example, which path would a monster affected by the Dissonant Whispers spell take?

Who operates delivery flights for commercial airlines?

Etymology of 'calcit(r)are'?



Finding the constrain of integral


ML-inequality for real integralsEvaluating the integral $int_0^infty fracsin x x ,mathrm dx = frac pi 2$?Proving the inequality of an integral applied to a sumFind values so that integral is a bounded operatorProblem in series expansionSolving the improper integral $1/(x^a+y^b)$Convergence of the integral: $I_alpha =int _0^infty left(frace^-alpha xsqrtxright):dx$definition of Fourier transform questionsHow to calculate $iint_D (x^2 + y^2) , dxdy$ with $D = (fracx2)^2 + (fracy3)^2 leq 1$?Integral transform with reciprocal complex exponential functions?permute double integral and limit













2












$begingroup$


$ f_n(x) = left(4x-x^2right)^n$



I'm trying to prove that $int _0^4:f_n(x),dx < 4^n+1$



When I'm doing a general integral to $n > 1$, I'm getting an expression that equals to zero when $X = 4$ or $X = 0$.



This is the expression:



$$fracleft(4x-x^2right)^n+1(4-2X)(n+1) $$



What am I missing? Because it doesn't not make sense that every integral with $n > 1$ is equal to zero










share|cite|improve this question











$endgroup$
















    2












    $begingroup$


    $ f_n(x) = left(4x-x^2right)^n$



    I'm trying to prove that $int _0^4:f_n(x),dx < 4^n+1$



    When I'm doing a general integral to $n > 1$, I'm getting an expression that equals to zero when $X = 4$ or $X = 0$.



    This is the expression:



    $$fracleft(4x-x^2right)^n+1(4-2X)(n+1) $$



    What am I missing? Because it doesn't not make sense that every integral with $n > 1$ is equal to zero










    share|cite|improve this question











    $endgroup$














      2












      2








      2


      1



      $begingroup$


      $ f_n(x) = left(4x-x^2right)^n$



      I'm trying to prove that $int _0^4:f_n(x),dx < 4^n+1$



      When I'm doing a general integral to $n > 1$, I'm getting an expression that equals to zero when $X = 4$ or $X = 0$.



      This is the expression:



      $$fracleft(4x-x^2right)^n+1(4-2X)(n+1) $$



      What am I missing? Because it doesn't not make sense that every integral with $n > 1$ is equal to zero










      share|cite|improve this question











      $endgroup$




      $ f_n(x) = left(4x-x^2right)^n$



      I'm trying to prove that $int _0^4:f_n(x),dx < 4^n+1$



      When I'm doing a general integral to $n > 1$, I'm getting an expression that equals to zero when $X = 4$ or $X = 0$.



      This is the expression:



      $$fracleft(4x-x^2right)^n+1(4-2X)(n+1) $$



      What am I missing? Because it doesn't not make sense that every integral with $n > 1$ is equal to zero







      analysis






      share|cite|improve this question















      share|cite|improve this question













      share|cite|improve this question




      share|cite|improve this question








      edited 9 hours ago









      Bernard

      127k743120




      127k743120










      asked 9 hours ago









      DanielDaniel

      111




      111




















          3 Answers
          3






          active

          oldest

          votes


















          4












          $begingroup$

          Hint:



          $4x-x^2$ is a quadratic polynomial , with a negative leading coefficient. Furthermore, it vanishes for $x=0$ and $x=4$. Can you find it (absolute) maximum?



          Once you've found this maximum, you can use the mean value inequality for integrals.






          share|cite|improve this answer









          $endgroup$












          • $begingroup$
            Of course, you can estimate this by the maximum! very nice approach. I like this argument. This is probably how the question was meant to be done ^^
            $endgroup$
            – Wesley Strik
            9 hours ago



















          1












          $begingroup$

          You cannot just blindly apply the reverse power rule. Remember the chain rule? You have a function within another function.



          Consider the case when n=2



          $ int_0^4 (4x + x^2)^2 dx = int_0^4 16x^2 + 8x^3 + x^4 dx$






          share|cite|improve this answer









          $endgroup$












          • $begingroup$
            Would it be better for him to use the binomial expansion, you think?
            $endgroup$
            – Wesley Strik
            9 hours ago






          • 1




            $begingroup$
            That would be an excellent approach. I was also thinking that you could attempt to find a pattern and then use induction to prove it.
            $endgroup$
            – agentnola
            9 hours ago










          • $begingroup$
            I started working out the binomial theorem approach, you can switch the order of integration and summation and then integrate the power of $x$, this should not be too hard, although it will get a bit messy: $$int _x=0^4:f_n(x),dx =int _x=0^4: (4x-x^2 )^n,dx =int _x=0^4: sum_k=0^n binomnk (4x)^k cdot (-x^2)^n-k,dx$$
            $endgroup$
            – Wesley Strik
            9 hours ago











          • $begingroup$
            Now reduce everything to some power of $x$, move the integral inside and then we hope to get something pretty?
            $endgroup$
            – Wesley Strik
            9 hours ago


















          0












          $begingroup$

          Below is a suggestion for a very tedious route, but I wanted to post it anyway. This is probably not how the question is meant to be done.



          Observe how by the binomial theorem we have that:
          $$I=int _x=0^4:f_n(x),dx =int _x=0^4: (4x-x^2 )^n,dx =int _x=0^4: sum_k=0^n binomnk (4x)^k cdot (-x^2)^n-k,dx$$
          We make this look a bit prettier:
          $$I=int _x=0^4: sum_k=0^n binomnk 4^k cdot (-1)^n-kx^2n-k,dx$$



          We may switch the order of integration and summation since this concerns polynomials (what I mean is that they are of finite order), we compute:
          $$I=sum_k=0^n binomnk frac12n-k+1 4^k cdot (-1)^n-k4^2n-k+1-0$$
          We collect some terms:
          $$I=sum_k=0^n binomnk frac12n-k+1 4^2n+1 cdot (-1)^n-k$$
          Using combinatorial formulae and some help from Wolfram Alpha, this can be exactly computed to be:
          $$ I =fracsqrt pi cdot n! cdot 2^2n+1(n+frac12)!=fracsqrt pi cdot n!(n+frac12)! cdot 4^n + frac12$$
          We then need to prove by induction that $fracsqrt pi cdot n!(n+frac12)! > sqrt4$ for positive $n$.



          Yes. This is a lot of hard work. You could also just take the approach that was posted by Bernard and observe that $4x-x^2$ has a global maximum on the interval $[0,4]$ (the begin and endpoint are zeros of this polynomial), this maximum by symmetry occurs at $x=2$. The function has maximum value $8-4=4$. Observe then by the estimation lemma that $M =max_x in [0,4]((4x-x^2)^n)=4^n$, and the length of our path is $4$. We get:
          $$ int _x=0^4 f_n dx leq M L = 4^n cdot 4 = 4^n+1 $$



          Also see: ML-inequality for real integrals






          share|cite|improve this answer









          $endgroup$













            Your Answer








            StackExchange.ready(function()
            var channelOptions =
            tags: "".split(" "),
            id: "69"
            ;
            initTagRenderer("".split(" "), "".split(" "), channelOptions);

            StackExchange.using("externalEditor", function()
            // Have to fire editor after snippets, if snippets enabled
            if (StackExchange.settings.snippets.snippetsEnabled)
            StackExchange.using("snippets", function()
            createEditor();
            );

            else
            createEditor();

            );

            function createEditor()
            StackExchange.prepareEditor(
            heartbeatType: 'answer',
            autoActivateHeartbeat: false,
            convertImagesToLinks: true,
            noModals: true,
            showLowRepImageUploadWarning: true,
            reputationToPostImages: 10,
            bindNavPrevention: true,
            postfix: "",
            imageUploader:
            brandingHtml: "Powered by u003ca class="icon-imgur-white" href="https://imgur.com/"u003eu003c/au003e",
            contentPolicyHtml: "User contributions licensed under u003ca href="https://creativecommons.org/licenses/by-sa/3.0/"u003ecc by-sa 3.0 with attribution requiredu003c/au003e u003ca href="https://stackoverflow.com/legal/content-policy"u003e(content policy)u003c/au003e",
            allowUrls: true
            ,
            noCode: true, onDemand: true,
            discardSelector: ".discard-answer"
            ,immediatelyShowMarkdownHelp:true
            );



            );













            draft saved

            draft discarded


















            StackExchange.ready(
            function ()
            StackExchange.openid.initPostLogin('.new-post-login', 'https%3a%2f%2fmath.stackexchange.com%2fquestions%2f3247910%2ffinding-the-constrain-of-integral%23new-answer', 'question_page');

            );

            Post as a guest















            Required, but never shown

























            3 Answers
            3






            active

            oldest

            votes








            3 Answers
            3






            active

            oldest

            votes









            active

            oldest

            votes






            active

            oldest

            votes









            4












            $begingroup$

            Hint:



            $4x-x^2$ is a quadratic polynomial , with a negative leading coefficient. Furthermore, it vanishes for $x=0$ and $x=4$. Can you find it (absolute) maximum?



            Once you've found this maximum, you can use the mean value inequality for integrals.






            share|cite|improve this answer









            $endgroup$












            • $begingroup$
              Of course, you can estimate this by the maximum! very nice approach. I like this argument. This is probably how the question was meant to be done ^^
              $endgroup$
              – Wesley Strik
              9 hours ago
















            4












            $begingroup$

            Hint:



            $4x-x^2$ is a quadratic polynomial , with a negative leading coefficient. Furthermore, it vanishes for $x=0$ and $x=4$. Can you find it (absolute) maximum?



            Once you've found this maximum, you can use the mean value inequality for integrals.






            share|cite|improve this answer









            $endgroup$












            • $begingroup$
              Of course, you can estimate this by the maximum! very nice approach. I like this argument. This is probably how the question was meant to be done ^^
              $endgroup$
              – Wesley Strik
              9 hours ago














            4












            4








            4





            $begingroup$

            Hint:



            $4x-x^2$ is a quadratic polynomial , with a negative leading coefficient. Furthermore, it vanishes for $x=0$ and $x=4$. Can you find it (absolute) maximum?



            Once you've found this maximum, you can use the mean value inequality for integrals.






            share|cite|improve this answer









            $endgroup$



            Hint:



            $4x-x^2$ is a quadratic polynomial , with a negative leading coefficient. Furthermore, it vanishes for $x=0$ and $x=4$. Can you find it (absolute) maximum?



            Once you've found this maximum, you can use the mean value inequality for integrals.







            share|cite|improve this answer












            share|cite|improve this answer



            share|cite|improve this answer










            answered 9 hours ago









            BernardBernard

            127k743120




            127k743120











            • $begingroup$
              Of course, you can estimate this by the maximum! very nice approach. I like this argument. This is probably how the question was meant to be done ^^
              $endgroup$
              – Wesley Strik
              9 hours ago

















            • $begingroup$
              Of course, you can estimate this by the maximum! very nice approach. I like this argument. This is probably how the question was meant to be done ^^
              $endgroup$
              – Wesley Strik
              9 hours ago
















            $begingroup$
            Of course, you can estimate this by the maximum! very nice approach. I like this argument. This is probably how the question was meant to be done ^^
            $endgroup$
            – Wesley Strik
            9 hours ago





            $begingroup$
            Of course, you can estimate this by the maximum! very nice approach. I like this argument. This is probably how the question was meant to be done ^^
            $endgroup$
            – Wesley Strik
            9 hours ago












            1












            $begingroup$

            You cannot just blindly apply the reverse power rule. Remember the chain rule? You have a function within another function.



            Consider the case when n=2



            $ int_0^4 (4x + x^2)^2 dx = int_0^4 16x^2 + 8x^3 + x^4 dx$






            share|cite|improve this answer









            $endgroup$












            • $begingroup$
              Would it be better for him to use the binomial expansion, you think?
              $endgroup$
              – Wesley Strik
              9 hours ago






            • 1




              $begingroup$
              That would be an excellent approach. I was also thinking that you could attempt to find a pattern and then use induction to prove it.
              $endgroup$
              – agentnola
              9 hours ago










            • $begingroup$
              I started working out the binomial theorem approach, you can switch the order of integration and summation and then integrate the power of $x$, this should not be too hard, although it will get a bit messy: $$int _x=0^4:f_n(x),dx =int _x=0^4: (4x-x^2 )^n,dx =int _x=0^4: sum_k=0^n binomnk (4x)^k cdot (-x^2)^n-k,dx$$
              $endgroup$
              – Wesley Strik
              9 hours ago











            • $begingroup$
              Now reduce everything to some power of $x$, move the integral inside and then we hope to get something pretty?
              $endgroup$
              – Wesley Strik
              9 hours ago















            1












            $begingroup$

            You cannot just blindly apply the reverse power rule. Remember the chain rule? You have a function within another function.



            Consider the case when n=2



            $ int_0^4 (4x + x^2)^2 dx = int_0^4 16x^2 + 8x^3 + x^4 dx$






            share|cite|improve this answer









            $endgroup$












            • $begingroup$
              Would it be better for him to use the binomial expansion, you think?
              $endgroup$
              – Wesley Strik
              9 hours ago






            • 1




              $begingroup$
              That would be an excellent approach. I was also thinking that you could attempt to find a pattern and then use induction to prove it.
              $endgroup$
              – agentnola
              9 hours ago










            • $begingroup$
              I started working out the binomial theorem approach, you can switch the order of integration and summation and then integrate the power of $x$, this should not be too hard, although it will get a bit messy: $$int _x=0^4:f_n(x),dx =int _x=0^4: (4x-x^2 )^n,dx =int _x=0^4: sum_k=0^n binomnk (4x)^k cdot (-x^2)^n-k,dx$$
              $endgroup$
              – Wesley Strik
              9 hours ago











            • $begingroup$
              Now reduce everything to some power of $x$, move the integral inside and then we hope to get something pretty?
              $endgroup$
              – Wesley Strik
              9 hours ago













            1












            1








            1





            $begingroup$

            You cannot just blindly apply the reverse power rule. Remember the chain rule? You have a function within another function.



            Consider the case when n=2



            $ int_0^4 (4x + x^2)^2 dx = int_0^4 16x^2 + 8x^3 + x^4 dx$






            share|cite|improve this answer









            $endgroup$



            You cannot just blindly apply the reverse power rule. Remember the chain rule? You have a function within another function.



            Consider the case when n=2



            $ int_0^4 (4x + x^2)^2 dx = int_0^4 16x^2 + 8x^3 + x^4 dx$







            share|cite|improve this answer












            share|cite|improve this answer



            share|cite|improve this answer










            answered 9 hours ago









            agentnolaagentnola

            389




            389











            • $begingroup$
              Would it be better for him to use the binomial expansion, you think?
              $endgroup$
              – Wesley Strik
              9 hours ago






            • 1




              $begingroup$
              That would be an excellent approach. I was also thinking that you could attempt to find a pattern and then use induction to prove it.
              $endgroup$
              – agentnola
              9 hours ago










            • $begingroup$
              I started working out the binomial theorem approach, you can switch the order of integration and summation and then integrate the power of $x$, this should not be too hard, although it will get a bit messy: $$int _x=0^4:f_n(x),dx =int _x=0^4: (4x-x^2 )^n,dx =int _x=0^4: sum_k=0^n binomnk (4x)^k cdot (-x^2)^n-k,dx$$
              $endgroup$
              – Wesley Strik
              9 hours ago











            • $begingroup$
              Now reduce everything to some power of $x$, move the integral inside and then we hope to get something pretty?
              $endgroup$
              – Wesley Strik
              9 hours ago
















            • $begingroup$
              Would it be better for him to use the binomial expansion, you think?
              $endgroup$
              – Wesley Strik
              9 hours ago






            • 1




              $begingroup$
              That would be an excellent approach. I was also thinking that you could attempt to find a pattern and then use induction to prove it.
              $endgroup$
              – agentnola
              9 hours ago










            • $begingroup$
              I started working out the binomial theorem approach, you can switch the order of integration and summation and then integrate the power of $x$, this should not be too hard, although it will get a bit messy: $$int _x=0^4:f_n(x),dx =int _x=0^4: (4x-x^2 )^n,dx =int _x=0^4: sum_k=0^n binomnk (4x)^k cdot (-x^2)^n-k,dx$$
              $endgroup$
              – Wesley Strik
              9 hours ago











            • $begingroup$
              Now reduce everything to some power of $x$, move the integral inside and then we hope to get something pretty?
              $endgroup$
              – Wesley Strik
              9 hours ago















            $begingroup$
            Would it be better for him to use the binomial expansion, you think?
            $endgroup$
            – Wesley Strik
            9 hours ago




            $begingroup$
            Would it be better for him to use the binomial expansion, you think?
            $endgroup$
            – Wesley Strik
            9 hours ago




            1




            1




            $begingroup$
            That would be an excellent approach. I was also thinking that you could attempt to find a pattern and then use induction to prove it.
            $endgroup$
            – agentnola
            9 hours ago




            $begingroup$
            That would be an excellent approach. I was also thinking that you could attempt to find a pattern and then use induction to prove it.
            $endgroup$
            – agentnola
            9 hours ago












            $begingroup$
            I started working out the binomial theorem approach, you can switch the order of integration and summation and then integrate the power of $x$, this should not be too hard, although it will get a bit messy: $$int _x=0^4:f_n(x),dx =int _x=0^4: (4x-x^2 )^n,dx =int _x=0^4: sum_k=0^n binomnk (4x)^k cdot (-x^2)^n-k,dx$$
            $endgroup$
            – Wesley Strik
            9 hours ago





            $begingroup$
            I started working out the binomial theorem approach, you can switch the order of integration and summation and then integrate the power of $x$, this should not be too hard, although it will get a bit messy: $$int _x=0^4:f_n(x),dx =int _x=0^4: (4x-x^2 )^n,dx =int _x=0^4: sum_k=0^n binomnk (4x)^k cdot (-x^2)^n-k,dx$$
            $endgroup$
            – Wesley Strik
            9 hours ago













            $begingroup$
            Now reduce everything to some power of $x$, move the integral inside and then we hope to get something pretty?
            $endgroup$
            – Wesley Strik
            9 hours ago




            $begingroup$
            Now reduce everything to some power of $x$, move the integral inside and then we hope to get something pretty?
            $endgroup$
            – Wesley Strik
            9 hours ago











            0












            $begingroup$

            Below is a suggestion for a very tedious route, but I wanted to post it anyway. This is probably not how the question is meant to be done.



            Observe how by the binomial theorem we have that:
            $$I=int _x=0^4:f_n(x),dx =int _x=0^4: (4x-x^2 )^n,dx =int _x=0^4: sum_k=0^n binomnk (4x)^k cdot (-x^2)^n-k,dx$$
            We make this look a bit prettier:
            $$I=int _x=0^4: sum_k=0^n binomnk 4^k cdot (-1)^n-kx^2n-k,dx$$



            We may switch the order of integration and summation since this concerns polynomials (what I mean is that they are of finite order), we compute:
            $$I=sum_k=0^n binomnk frac12n-k+1 4^k cdot (-1)^n-k4^2n-k+1-0$$
            We collect some terms:
            $$I=sum_k=0^n binomnk frac12n-k+1 4^2n+1 cdot (-1)^n-k$$
            Using combinatorial formulae and some help from Wolfram Alpha, this can be exactly computed to be:
            $$ I =fracsqrt pi cdot n! cdot 2^2n+1(n+frac12)!=fracsqrt pi cdot n!(n+frac12)! cdot 4^n + frac12$$
            We then need to prove by induction that $fracsqrt pi cdot n!(n+frac12)! > sqrt4$ for positive $n$.



            Yes. This is a lot of hard work. You could also just take the approach that was posted by Bernard and observe that $4x-x^2$ has a global maximum on the interval $[0,4]$ (the begin and endpoint are zeros of this polynomial), this maximum by symmetry occurs at $x=2$. The function has maximum value $8-4=4$. Observe then by the estimation lemma that $M =max_x in [0,4]((4x-x^2)^n)=4^n$, and the length of our path is $4$. We get:
            $$ int _x=0^4 f_n dx leq M L = 4^n cdot 4 = 4^n+1 $$



            Also see: ML-inequality for real integrals






            share|cite|improve this answer









            $endgroup$

















              0












              $begingroup$

              Below is a suggestion for a very tedious route, but I wanted to post it anyway. This is probably not how the question is meant to be done.



              Observe how by the binomial theorem we have that:
              $$I=int _x=0^4:f_n(x),dx =int _x=0^4: (4x-x^2 )^n,dx =int _x=0^4: sum_k=0^n binomnk (4x)^k cdot (-x^2)^n-k,dx$$
              We make this look a bit prettier:
              $$I=int _x=0^4: sum_k=0^n binomnk 4^k cdot (-1)^n-kx^2n-k,dx$$



              We may switch the order of integration and summation since this concerns polynomials (what I mean is that they are of finite order), we compute:
              $$I=sum_k=0^n binomnk frac12n-k+1 4^k cdot (-1)^n-k4^2n-k+1-0$$
              We collect some terms:
              $$I=sum_k=0^n binomnk frac12n-k+1 4^2n+1 cdot (-1)^n-k$$
              Using combinatorial formulae and some help from Wolfram Alpha, this can be exactly computed to be:
              $$ I =fracsqrt pi cdot n! cdot 2^2n+1(n+frac12)!=fracsqrt pi cdot n!(n+frac12)! cdot 4^n + frac12$$
              We then need to prove by induction that $fracsqrt pi cdot n!(n+frac12)! > sqrt4$ for positive $n$.



              Yes. This is a lot of hard work. You could also just take the approach that was posted by Bernard and observe that $4x-x^2$ has a global maximum on the interval $[0,4]$ (the begin and endpoint are zeros of this polynomial), this maximum by symmetry occurs at $x=2$. The function has maximum value $8-4=4$. Observe then by the estimation lemma that $M =max_x in [0,4]((4x-x^2)^n)=4^n$, and the length of our path is $4$. We get:
              $$ int _x=0^4 f_n dx leq M L = 4^n cdot 4 = 4^n+1 $$



              Also see: ML-inequality for real integrals






              share|cite|improve this answer









              $endgroup$















                0












                0








                0





                $begingroup$

                Below is a suggestion for a very tedious route, but I wanted to post it anyway. This is probably not how the question is meant to be done.



                Observe how by the binomial theorem we have that:
                $$I=int _x=0^4:f_n(x),dx =int _x=0^4: (4x-x^2 )^n,dx =int _x=0^4: sum_k=0^n binomnk (4x)^k cdot (-x^2)^n-k,dx$$
                We make this look a bit prettier:
                $$I=int _x=0^4: sum_k=0^n binomnk 4^k cdot (-1)^n-kx^2n-k,dx$$



                We may switch the order of integration and summation since this concerns polynomials (what I mean is that they are of finite order), we compute:
                $$I=sum_k=0^n binomnk frac12n-k+1 4^k cdot (-1)^n-k4^2n-k+1-0$$
                We collect some terms:
                $$I=sum_k=0^n binomnk frac12n-k+1 4^2n+1 cdot (-1)^n-k$$
                Using combinatorial formulae and some help from Wolfram Alpha, this can be exactly computed to be:
                $$ I =fracsqrt pi cdot n! cdot 2^2n+1(n+frac12)!=fracsqrt pi cdot n!(n+frac12)! cdot 4^n + frac12$$
                We then need to prove by induction that $fracsqrt pi cdot n!(n+frac12)! > sqrt4$ for positive $n$.



                Yes. This is a lot of hard work. You could also just take the approach that was posted by Bernard and observe that $4x-x^2$ has a global maximum on the interval $[0,4]$ (the begin and endpoint are zeros of this polynomial), this maximum by symmetry occurs at $x=2$. The function has maximum value $8-4=4$. Observe then by the estimation lemma that $M =max_x in [0,4]((4x-x^2)^n)=4^n$, and the length of our path is $4$. We get:
                $$ int _x=0^4 f_n dx leq M L = 4^n cdot 4 = 4^n+1 $$



                Also see: ML-inequality for real integrals






                share|cite|improve this answer









                $endgroup$



                Below is a suggestion for a very tedious route, but I wanted to post it anyway. This is probably not how the question is meant to be done.



                Observe how by the binomial theorem we have that:
                $$I=int _x=0^4:f_n(x),dx =int _x=0^4: (4x-x^2 )^n,dx =int _x=0^4: sum_k=0^n binomnk (4x)^k cdot (-x^2)^n-k,dx$$
                We make this look a bit prettier:
                $$I=int _x=0^4: sum_k=0^n binomnk 4^k cdot (-1)^n-kx^2n-k,dx$$



                We may switch the order of integration and summation since this concerns polynomials (what I mean is that they are of finite order), we compute:
                $$I=sum_k=0^n binomnk frac12n-k+1 4^k cdot (-1)^n-k4^2n-k+1-0$$
                We collect some terms:
                $$I=sum_k=0^n binomnk frac12n-k+1 4^2n+1 cdot (-1)^n-k$$
                Using combinatorial formulae and some help from Wolfram Alpha, this can be exactly computed to be:
                $$ I =fracsqrt pi cdot n! cdot 2^2n+1(n+frac12)!=fracsqrt pi cdot n!(n+frac12)! cdot 4^n + frac12$$
                We then need to prove by induction that $fracsqrt pi cdot n!(n+frac12)! > sqrt4$ for positive $n$.



                Yes. This is a lot of hard work. You could also just take the approach that was posted by Bernard and observe that $4x-x^2$ has a global maximum on the interval $[0,4]$ (the begin and endpoint are zeros of this polynomial), this maximum by symmetry occurs at $x=2$. The function has maximum value $8-4=4$. Observe then by the estimation lemma that $M =max_x in [0,4]((4x-x^2)^n)=4^n$, and the length of our path is $4$. We get:
                $$ int _x=0^4 f_n dx leq M L = 4^n cdot 4 = 4^n+1 $$



                Also see: ML-inequality for real integrals







                share|cite|improve this answer












                share|cite|improve this answer



                share|cite|improve this answer










                answered 8 hours ago









                Wesley StrikWesley Strik

                2,310424




                2,310424



























                    draft saved

                    draft discarded
















































                    Thanks for contributing an answer to Mathematics Stack Exchange!


                    • Please be sure to answer the question. Provide details and share your research!

                    But avoid


                    • Asking for help, clarification, or responding to other answers.

                    • Making statements based on opinion; back them up with references or personal experience.

                    Use MathJax to format equations. MathJax reference.


                    To learn more, see our tips on writing great answers.




                    draft saved


                    draft discarded














                    StackExchange.ready(
                    function ()
                    StackExchange.openid.initPostLogin('.new-post-login', 'https%3a%2f%2fmath.stackexchange.com%2fquestions%2f3247910%2ffinding-the-constrain-of-integral%23new-answer', 'question_page');

                    );

                    Post as a guest















                    Required, but never shown





















































                    Required, but never shown














                    Required, but never shown












                    Required, but never shown







                    Required, but never shown

































                    Required, but never shown














                    Required, but never shown












                    Required, but never shown







                    Required, but never shown







                    Popular posts from this blog

                    Invision Community Contents History See also References External links Navigation menuProprietaryinvisioncommunity.comIPS Community ForumsIPS Community Forumsthis blog entry"License Changes, IP.Board 3.4, and the Future""Interview -- Matt Mecham of Ibforums""CEO Invision Power Board, Matt Mecham Is a Liar, Thief!"IPB License Explanation 1.3, 1.3.1, 2.0, and 2.1ArchivedSecurity Fixes, Updates And Enhancements For IPB 1.3.1Archived"New Demo Accounts - Invision Power Services"the original"New Default Skin"the original"Invision Power Board 3.0.0 and Applications Released"the original"Archived copy"the original"Perpetual licenses being done away with""Release Notes - Invision Power Services""Introducing: IPS Community Suite 4!"Invision Community Release Notes

                    Canceling a color specificationRandomly assigning color to Graphics3D objects?Default color for Filling in Mathematica 9Coloring specific elements of sets with a prime modified order in an array plotHow to pick a color differing significantly from the colors already in a given color list?Detection of the text colorColor numbers based on their valueCan color schemes for use with ColorData include opacity specification?My dynamic color schemes

                    Tom Holland Mục lục Đầu đời và giáo dục | Sự nghiệp | Cuộc sống cá nhân | Phim tham gia | Giải thưởng và đề cử | Chú thích | Liên kết ngoài | Trình đơn chuyển hướngProfile“Person Details for Thomas Stanley Holland, "England and Wales Birth Registration Index, 1837-2008" — FamilySearch.org”"Meet Tom Holland... the 16-year-old star of The Impossible""Schoolboy actor Tom Holland finds himself in Oscar contention for role in tsunami drama"“Naomi Watts on the Prince William and Harry's reaction to her film about the late Princess Diana”lưu trữ"Holland and Pflueger Are West End's Two New 'Billy Elliots'""I'm so envious of my son, the movie star! British writer Dominic Holland's spent 20 years trying to crack Hollywood - but he's been beaten to it by a very unlikely rival"“Richard and Margaret Povey of Jersey, Channel Islands, UK: Information about Thomas Stanley Holland”"Tom Holland to play Billy Elliot""New Billy Elliot leaving the garage"Billy Elliot the Musical - Tom Holland - Billy"A Tale of four Billys: Tom Holland""The Feel Good Factor""Thames Christian College schoolboys join Myleene Klass for The Feelgood Factor""Government launches £600,000 arts bursaries pilot""BILLY's Chapman, Holland, Gardner & Jackson-Keen Visit Prime Minister""Elton John 'blown away' by Billy Elliot fifth birthday" (video with John's interview and fragments of Holland's performance)"First News interviews Arrietty's Tom Holland"“33rd Critics' Circle Film Awards winners”“National Board of Review Current Awards”Bản gốc"Ron Howard Whaling Tale 'In The Heart Of The Sea' Casts Tom Holland"“'Spider-Man' Finds Tom Holland to Star as New Web-Slinger”lưu trữ“Captain America: Civil War (2016)”“Film Review: ‘Captain America: Civil War’”lưu trữ“‘Captain America: Civil War’ review: Choose your own avenger”lưu trữ“The Lost City of Z reviews”“Sony Pictures and Marvel Studios Find Their 'Spider-Man' Star and Director”“‘Mary Magdalene’, ‘Current War’ & ‘Wind River’ Get 2017 Release Dates From Weinstein”“Lionsgate Unleashing Daisy Ridley & Tom Holland Starrer ‘Chaos Walking’ In Cannes”“PTA's 'Master' Leads Chicago Film Critics Nominations, UPDATED: Houston and Indiana Critics Nominations”“Nominaciones Goya 2013 Telecinco Cinema – ENG”“Jameson Empire Film Awards: Martin Freeman wins best actor for performance in The Hobbit”“34th Annual Young Artist Awards”Bản gốc“Teen Choice Awards 2016—Captain America: Civil War Leads Second Wave of Nominations”“BAFTA Film Award Nominations: ‘La La Land’ Leads Race”“Saturn Awards Nominations 2017: 'Rogue One,' 'Walking Dead' Lead”Tom HollandTom HollandTom HollandTom Hollandmedia.gettyimages.comWorldCat Identities300279794no20130442900000 0004 0355 42791085670554170004732cb16706349t(data)XX5557367